You are on page 1of 11

Multiple-Choice Test

Simpson’s 1/3 Rule


Integration
COMPLETE SOLUTION SET

1. The highest order of polynomial integrand for which Simpson’s 1/3 rule of integration is
exact is
(A) first
(B) second
(C) third
(D) fourth

Solution
The correct answer is (C).

Simpson’s 1/3 rule of integration is exact for integrating polynomials of third order or less.
Although Simpson’s 1/3 rule is derived by approximating the integrand by a second order
polynomial, the area under the curve is exact for a third order polynomial. Without proof it can
(b − a ) 5 ( 4)
be shown that the truncation error in Simpson’s 1/3 rule is E t = − f (ζ ), a < ζ < b .
2880
Since the fourth derivative of a third order polynomial is zero, the truncation error would be zero.
Hence Simpson’s 1/3 rule is exact for integrating polynomials of third order or less.
2.2

∫e
x
2. The value of dx by using 2-segment Simpson’s 1/3 rule most nearly is
0.2

(A) 7.8036
(B) 7.8423
(C) 8.4433
(D) 10.246

Solution
The correct answer is (B).

The multiple segment equation for Simpson’s 1/3 rule is


b ⎡ ⎤
b−a⎢ n −1 n−2

∫a f ( x)dx ≈ 3n ⎢ f ( x0 ) + 4 ∑ f ( xi ) + 2 ∑ f ( x i ) + f ( x n ) ⎥
i =1 i =2

⎢⎣ i = odd i = even ⎥⎦
Using two-segments gives

a = 0.2
b = 2.2
n=2
b−a
h=
n
2.2 − 0.2
=
2
=1
x0 = 0.2
x1 = x0 + h
= 0.2 + 1
= 1.2
x2 = 1.2 + 1
= 2.2
2.2 ⎡ ⎤
2.2 − 0.2 ⎢ 2 −1 2−2

∫0.2e dx ≈ 3 × 2 ⎢ f (0.2) + 4 ∑
x
f ( xi ) + 2 ∑ f ( xi ) + f (2.2)⎥
i =1 i =2

⎢⎣ i =odd i =even ⎥⎦
⎡ ⎤
2.2 − 0.2 ⎢ 1 0
= f (0.2) + 4 ∑ f ( xi ) + 2 ∑ f ( xi ) + f (2.2)⎥
3× 2 ⎢ i =1 i =2

⎣⎢ i =odd i =even ⎦⎥
2.2 − 0.2
= [ f (0.2) + 4 f (1.2) + f (2.2)]
3× 2
[
= 0.33333 e 0.2 + 4 × e1.2 + e 2.2 ]
= 0.33333[23.527]
= 7.8423
2.2

∫e
x
3. The value of dx by using 4-segment Simpson’s 1/3 rule most nearly is
0.2

(A) 7.8036
(B) 7.8062
(C) 7.8423
(D) 7.9655

Solution
The correct answer is (B).

The multiple segment equation for Simpson’s 1/3 rule is


b ⎡ ⎤
b−a ⎢ n −1 n−2

∫a f ( x)dx ≈ 3n ⎢ f ( x0 ) + 4 ∑ f ( xi ) + 2 ∑ f ( xi ) + f ( xn )⎥
i =1 i =2

⎣⎢ i =odd i =even ⎦⎥

Using 4 segments gives


a = 0.2
b = 2.2
n=4
b−a
h=
n
2.2 − 0.2
=
4
= 0.5
2.2 ⎡ ⎤
2.2 − 0.2 ⎢ 4−1 4−2

∫0.2e dx ≈ 3 × 4 ⎢ f ( x0 ) + 4 ∑
x
f ( xi ) + 2 ∑ f ( xi ) + f ( x4 )⎥
i =1 i =2

⎣⎢ i =odd i =even ⎦⎥
⎡ ⎤
2.2 − 0.2 ⎢ 3 2
= f ( x0 ) + 4 ∑ f ( xi ) + 2 ∑ f ( xi ) + f ( x4 )⎥
3× 4 ⎢ i =1 i =2

⎢⎣ i =odd i =even ⎥⎦
2.2 − 0.2
= [ f ( x0 ) + 4( f (x1 ) + f (x3 )) + 2( f (x2 )) + f ( x4 )]
3× 4
So
f (x ) = e x
f ( x0 ) = f (0.2) = e 0.2 = 1.2214

f ( x1 ) = f (0.2 + 0.5) = f (0.7 )


f (0.7 ) = e 0.7 = 2.0138

f ( x2 ) = f (0.7 + 0.5) = f (1.2)


f (1.2) = e1.2 = 3.3201

f ( x3 ) = f (1.2 + 0.5) = f (1.7 )


f (1.7 ) = e1.7 = 5.4739

f ( x4 ) = f (2.2) = e 2.2 = 9.0250

2.2
2.2 − 0.2
∫ e dx ≈
x
[ f ( x0 ) + 4( f (x1 ) + f (x3 )) + 2( f (x2 )) + f ( x4 )]
0.2
3× 4
2.2 − 0.2
= [1.2214 + 4(2.0138 + 5.4739) + 2(3.3201) + 9.0250]
3× 4
= 0.16667[1.2214 + 29.951 + 6.6402 + 9.0250]
= 7.8062
4. The velocity of a body is given by
v(t ) = 2t , 1≤ t ≤ 5
= 5t 2 + 3, 5 < t ≤ 14
where t is given in seconds, and v is given in m/s. Using two-segment Simpson’s
1/3 rule, the distance in meters covered by the body from t = 2 to t = 9 seconds most
nearly is
(A) 949.33
(B) 1039.7
(C) 1200.5
(D) 1442.0

Solution
The correct answer is (C).

The multiple segment equation for Simpson’s 1/3 rule is


b ⎡ ⎤
b−a ⎢ n −1 n−2

∫a f ( x)dx ≈ 3n ⎢ f ( x0 ) + 4 ∑ f ( xi ) + 2 ∑ f ( xi ) + f ( xn )⎥
i =1 i =2

⎢⎣ i =odd i =even ⎥⎦

a=2
b=9
n=2
b−a
h=
n
9−2
=
2
= 3.5
9 ⎡ ⎤
9−2⎢ 2−1 2− 2

∫2 v(t )dt ≈ 3 × 2 ⎢v(t0 ) + 4 ∑ v(ti ) + 2 ∑ v(ti ) + v(t 2 )⎥


i =1 i =2

⎣⎢ i =odd i =even ⎦⎥
⎡ ⎤
9−2⎢ 1 0
= v(t0 ) + 4 ∑ v(ti ) + 2 ∑ v(ti ) + v(t 2 )⎥
3× 2 ⎢ i =1 i =2

⎢⎣ i =odd i =even ⎥⎦
9−2
= [v(t0 ) + 4(v(t1 )) + v(t2 )]
3× 2
So
v(t ) = 2t , 1≤ t ≤ 5
= 5t 2 + 3, 5 < t ≤ 14

v(t0 ) = v(2) = 2 × 2 = 4 m/s

v(t1 ) = v(2 + 3.5) = v(5.5)


v(5.5) = 5 × 5.52 + 3 = 154.25 m/s

v(t 2 ) = v(9) = 5 × 9 2 + 3 = 408 m/s

9
9−2
∫ v(t )dt ≈ 3 × 2 [v(t ) + 4 × v(t ) + v(t )]
2
0 1 2

9−2
= [v(2) + 4 × v(5.5) + v(9)]
3× 2
= 1.1667[4 + 4 × 154.25 + 408]
= 1200.5 m
19
5. The value of ∫ f ( x)dx by using 2-segment Simpson’s 1/3 rule is estimated as
3

702.039. The estimate of the same integral using 4-segment Simpson’s 1/3 rule most
nearly is
8
(A) 702.039 + [2 f (7 ) − f (11) + 2 f (15)]
3
702.039 8
(B) + [2 f (7 ) − f (11) + 2 f (15)]
2 3
8
(C) 702.039 + [2 f (7 ) + 2 f (15)]
3
702.039 8
(D) + [2 f (7 )2 f (15)]
2 3
Solution
The correct answer is (B).

Using 2-segment Simpson’s 1/3 rule gives


19 ⎡ ⎤
19 − 3 ⎢ 2−1 2− 2

∫3 f ( x)dx ≈ 3 × 2 ⎢ f ( x0 ) + 4 ∑ f ( xi ) + 2 ∑ f ( xi ) + f ( x2 )⎥
i =1 i =2

⎣⎢ i =odd i =even ⎦⎥
⎡ ⎤
19 − 3 ⎢ 1
= f ( x0 ) + 4 ∑ f ( xi ) + f ( x2 )⎥
3× 2 ⎢ i =1

⎢⎣ i =odd ⎥⎦
19 − 3
= [ f ( x0 ) + 4 f (x1 ) + f ( x2 )]
3× 2
19 − 3
702.039 ≈ [ f (3) + 4 f (11) + f (19)]
3× 2
Using 4-segment Simpson’s 1/3 rule gives
19 ⎡ ⎤
19 − 3 ⎢ 4−1 4− 2

∫3 f ( x)dx ≈ 3 × 4 ⎢ f ( x0 ) + 4 ∑ f ( xi ) + 2 ∑ f ( xi ) + f ( x4 )⎥
i =1 i =2

⎢⎣ i =odd i =even ⎥⎦
⎡ ⎤
19 − 3 ⎢ 3 2
= f ( x0 ) + 4 ∑ f ( xi ) + 2 ∑ f ( xi ) + f ( x4 )⎥
3× 4 ⎢ i =1 i =2

⎣⎢ i =odd i =even ⎦⎥
19 − 3
= [ f ( x0 ) + 4( f (x1 ) + f (x3 )) + 2( f (x2 )) + f ( x4 )]
3× 4
19 − 3
= [ f (3) + 4( f (7 ) + f (15)) + 2( f (11)) + f (19)]
3× 4
19 − 3
= [ f (3) + 4 f (7 ) + 4 f (11) + 4 f (15) − 2 f (11) + f (19)]
3× 4
19 − 3
= [ f (3) + 4 f (11) + f (19)] + 19 − 3 [4 f (7 ) + 4 f (15) − 2 f (11)]
3× 4 3× 4
702.039 19 − 3
= + [4 f (7 ) + 4 f (15) − 2 f (11)]
2 3× 4
702.039 2(19 − 3)
= + [2 f (7 ) + 2 f (15) − f (11)]
2 3× 4
702.039 8
= + [2 f (7 ) + 2 f (15) − f (11)]
2 3
6. The following data of the velocity of a body is given as a function of time.
Time (s) 4 7 10 15
Velocity (m/s) 22 24 37 46
The best estimate of the distance in meters covered by the body from t = 4 to t = 15
using combined Simpson’s 1/3 rule and the trapezoidal rule would be
(A) 354.70
(B) 362.50
(C) 368.00
(D) 378.80

Solution
The correct answer is (B).

t0 = 4, t1 = 7, t2 = 10, t3 = 15
We can use Simpson’s 1/3 rule from t = 4 to t = 10 as we have three equidistant points,
t = 4,7,10.
b ⎡ ⎤
b−a ⎢ n −1 n−2

∫a v(t )dt ≈ 3n ⎢v(t0 ) + 4 ∑ v(ti ) + 2 ∑ v(ti ) + v(t n )⎥


i =1 i =2

⎢⎣ i =odd i =even ⎥⎦
where
a = t0
b = t2
h = t 2 − t1
= t1 − t0
=3
b−a
h=
n
10 − 4
3=
n
n=2
Thus, using 2-segment Simpson’s 1/3 rule
t2
t −t
∫t v(t )dt ≈ 32 × 20 [v(t0 ) + 4v(t1 ) + v(t2 )]
0

Using the trapezoidal rule with unequal segments from t = 10 to t = 15


⎡ v(t ) + v(t )⎤
t3

∫t v(t )dt ≈ (t3 − t2 )⎢⎣ 2 2 3 ⎥⎦


2
Thus,
t3

∫ v(t )dt ≈
t 2 − t0
[v(t0 ) + 4v(t1 ) + v(t2 )] + (t3 − t2 ) ⎡⎢ v(t2 ) + v(t3 ) ⎤⎥
t0
3× 2 ⎣ 2 ⎦
14

∫ v(t )dt ≈
10 − 4
[v(4) + 4v(7 ) + v(10)] + (15 − 10) ⎡⎢ v(10) + v(15) ⎤⎥
4
3× 2 ⎣ 2 ⎦

=
10 − 4
[22 + 4 × 24 + 37] + (15 − 10) ⎡⎢ 37 + 46 ⎤⎥
3× 2 ⎣ 2 ⎦
= (6)[25.833] + (5)[41.5]
= 362.5 m

You might also like